Udowadnianie wymiarów podprzestrzeni?

Przestrzenie wektorowe, bazy, liniowa niezależność, macierze.... Formy kwadratowe, twierdzenia o klasyfikacji...
Drelson
Użytkownik
Użytkownik
Posty: 79
Rejestracja: 7 paź 2014, o 18:01
Płeć: Mężczyzna
Lokalizacja: warszawa
Podziękował: 16 razy

Udowadnianie wymiarów podprzestrzeni?

Post autor: Drelson »

Witam , czy ktoś z Was by mógł mi napisać o co chodzi i jak się zabrać do tego zadania:

Niech \(\displaystyle{ \mathrm V}\) będzie przestrzenią liniową \(\displaystyle{ \mathbb K^{n}}\) nad ciałem \(\displaystyle{ \mathbb K}\) oraz niech \(\displaystyle{ \mathrm W_{1}}\) i \(\displaystyle{ \mathrm W_{2}}\) będą jej podprzestrzeniami. Zdefiniujmy zbiór \(\displaystyle{ \mathrm W_{1} + \mathrm W_{2} = \left\{ \mathrm w_{1} + \mathrm w_{2} : \mathrm w_{1} \in \mathrm W_{1} , \mathrm w_{2} \in \mathrm W_{2} \right\}}\) Udowodnić, że :

a) Jeśli \(\displaystyle{ \mathrm dim\left( \mathrm W_{1} + \mathrm W_{2}\right) = 1+ \mathrm dim\left( \mathrm W_{1} \cap \mathrm W_{2}\right)}\) , to \(\displaystyle{ \mathrm W_{1} + \mathrm W_{2}}\) jest równa jednej z tych podprzestrzeni a przecięcie \(\displaystyle{ \mathrm W_{1} \cap \mathrm W_{2}}\) - drugiej

b) Jeśli \(\displaystyle{ \mathrm dim \mathrm W_{1} + \mathrm dim \mathrm W_{2} > \mathrm dim V = \mathrm n}\), to \(\displaystyle{ \mathrm W_{1} \cap \mathrm W_{2} \neq \left\{ 0\right\}}\)
Awatar użytkownika
sebnorth
Użytkownik
Użytkownik
Posty: 635
Rejestracja: 12 sty 2011, o 16:27
Płeć: Mężczyzna
Lokalizacja: Puck i Trójmiasto
Pomógł: 201 razy

Udowadnianie wymiarów podprzestrzeni?

Post autor: sebnorth »

oznaczmy \(\displaystyle{ d_1 = \dim W_1, d_2 = \dim W_2, d_{12} = W_1 \cap W_2, d_{1+2} =W_1 + W_2}\)

Oczywiście: \(\displaystyle{ W_1 \cap W_2 \subset W_1 \subset W_1 + W_2}\)

czyli \(\displaystyle{ d_{12} \leq d_1 \leq d_{1+2} = 1 + d_{12}}\)

Zatem są dwie opcje:

1) \(\displaystyle{ d_{12} = d_1}\), czyli \(\displaystyle{ W_1 \cap W_2 = W_1, W_1 \subset W_2, W_1 + W_2 = W_2}\)

2) \(\displaystyle{ d_1 = d_{1+2}}\) czyli \(\displaystyle{ W_1 = W_1 + W_2, W_2 \subset W_1, W_2 \cap W_1 = W_2}\)

b)

\(\displaystyle{ n < d_1 + d_2 = d_{1+2} + d_{12} \leq n + d_{12}}\)

\(\displaystyle{ 0 < d_{12}}\)

zatem \(\displaystyle{ \mathrm W_{1} \cap \mathrm W_{2} \neq \left\{ 0\right\}}\)
ODPOWIEDZ